LSAT and Law School Admissions Forum

Get expert LSAT preparation and law school admissions advice from PowerScore Test Preparation.

User avatar
 Dave Killoran
PowerScore Staff
  • PowerScore Staff
  • Posts: 5853
  • Joined: Mar 25, 2011
|
#49163
Complete Question Explanation
(The complete setup for this game can be found here: lsat/viewtopic.php?t=9142)

The correct answer choice is (E)

Answer choice (A) is incorrect because F and V cannot be prescribed together.

Answer choice (B) is incorrect because F and V cannot be prescribed together, and also because two dietary regimens cannot be prescribed.

Answer choice (C) is incorrect because F and M cannot be prescribed together, and also because two dietary regimens cannot be prescribed.

Answer choice (D) is incorrect because from the fifth rule if N and U are prescribed, then G cannot be prescribed.

Answer choice (E) is the correct answer choice.
User avatar
 LeslyLSATinLA
  • Posts: 16
  • Joined: Jan 30, 2023
|
#99360
I'm still confused on why A is wrong E is correct. Can someone continue explaining? The explanation for answer choice B says, "also because two dietary regimens cannot be prescribed." would this also apply to answer choice A, but V is a phys not a diet.
 Adam Tyson
PowerScore Staff
  • PowerScore Staff
  • Posts: 5153
  • Joined: Apr 14, 2011
|
#99370
The second rule tells us that if you do not have O in the group, you cannot have F in the group:

O :arrow: F

(The font here may make it hard to see, but the F in the necessary condition above is crossed out)

The contrapositive of that rule is a little easier on the brain, in my opinion, and will make clear why answer A is a problem. That contrapositive is:

F :arrow: O

If F is selected, then O must also be selected, but answer A has F and does not have O, violating that rule.

Get the most out of your LSAT Prep Plus subscription.

Analyze and track your performance with our Testing and Analytics Package.